Sunteți pe pagina 1din 23

1

Constantino vs. Asia Life Insurance Co. [GR# L-1669


August 31, 1950] Peralta vs. Asia Life Insurance Co.
[GR# L-1670 August 31, 1950]
Facts: FIRST CASE: Respondent Corporation was paid P 176.04
as annual premium by Arcadio Constantino in exchange for
policy no. 93212 on 1941 for P 3,000 which lasted for 20
years. Petitioner Paz Constantino was made beneficiary.
However after the first payment, no further premiums were
made. Thereafter the insured died on 1944. Later, due to the
war (Japanese occupation) Respondent Corporation had to
close down its branch in the country.
SECOND CASE: Similarly, Respondent Corporation issued on
1938 another insurance policy no. 78145 for Spouses Ruiz and
Peralta also for P 3,000, lasting for 20 years. Regular
payments were made however due also to the war, it became
impossible to transact further payments. The insured
nevertheless was able to borrow P 234 from the policy. Ruiz
died on 1945. Peralta was the beneficiary.
In both cases the plaintiffs demanded payment but was
refused due to Respondent Corporations refusal on the
ground of non-payment of the premiums. The lower court
favored Respondent.

was hold that promptness of payment is essential in the


business of life insurance since all calculations of the
company is based on the hypothesis of prompt payments.
Forfeiture for non-payment is necessary to protect said
business from embarrassment otherwise confusion would
abound. And that delinquency cannot be tolerated nor
redeemed except at the option of the company. Lastly parties
contracted both for peace and war times since the policies
contained also wartime days. It follows that the parties
contemplated uninterrupted operation of the contract even if
armed conflict ensues.

(2) The annual premium is not a debt, nor is it an obligation


which the insurer can maintain an action against the insured;
nor its settlement governed by the rules on payment of
debts.

A contract of insurance is sui generis. This means though the


insured may hold the insurer to the contract by the fulfillment
of the condition, the latter has no power or right to compel the
insured to maintain the contract relation longer than the
insured may desire. It is optional upon the insured.

Issues:
(1) Whether or not the beneficiaries are entitled to recover the
amount insured despite non-payment caused by the Japanese
occupation.
(2) Whether or not the periodic payments of the premiums,
those after the first, is not an obligation of the insured so that
it is not a debt enforceable by the action of the insurer.
Held:
(1) The beneficiaries are not entitled to recover for nonpayment despite the presence of war.
Contracts of insurance are contracts of indemnity within the
terms and condition found therein. An insurance company for
certain considerations guarantee the insured against loss or
damage as may be stipulated, and when called to pay, the
insurer may insist on the fulfillment of said stipulations. Failure
of the insured to do so disqualifies recovery for the loss. Thus
the terms of the policy determines the insurers liability.
Compliance to the terms of the policy is a must as it is a
condition precedent to the right of recovery. Therefore, from
the terms of the policy it is clear that non-payment of
premium produces avoidance (forfeiture of the policy).
Moreover, since act 2427, Philippine law on insurance and the
Civil Code) are mostly based from the Civil Code of California,
An intention to supplement our laws with the prevailing
principles of the US arises. Thus, Prof. Vance of Yaled declares
that the United States Rule must be followed, where the
contract is not merely suspended but is abrogated by reason
of non-payment of premiums since the time of payments is
peculiar to the essence of the contract. Further it would be
unjust to permit the insurer to retain the reserve value of the
policy or the excess of premiums paid over the actual risk
when the policy was still effective as held in the Statham Case
which was more logical and juridically sound. In said case it

INSULAR LIFE ASSURANCE COMPANY, LTD. vs.


CARPONIA T. EBRADO
FACTS: On September 1, 1968, Buenaventura Cristor Ebrado
was issued by The Life Assurance Co., Ltd., on a whole-life for
P5,882.00 with a rider for Accidental Death for the same
amount. He designated Carponia T. Ebrado, his common-law
wife as the revocable beneficiary in his policy. He referred to
her as his wife in the policy. On October 21, 1969, He died as
a result of an accident when he was hit by a failing branch of
a tree. As the policy was in force, the insurance company was
liable to pay the coverage in the total amount of P11,745.73,
representing the face value of the policy in the amount of
P5,882.00 plus the additional benefits for accidental death
also in the amount of P5,882.00 and the refund of P18.00 paid
for the premium due November, 1969, minus the unpaid
premiums and interest thereon due for January and February,
1969, in the sum of P36.27. Carponia T. Ebrado filed a claim
for the proceeds of the Policy as the designated beneficiary
therein, although she admits that she and the insured
Buenaventura C. Ebrado were merely living as husband and
wife without the benefit of marriage. Pascual T. Ebrado, also
filed a claim to the insurance company, this time claiming to
be the legal wife Buenaventura. She asserts that she has a
better right over the proceeds than Carponia who is a
common-law wife. As the insurance company is at a loss as to
whom to give the proceeds, it commenced an action for
interpleader in court. After the issues have been joined, a pretrial conference was held on July 8, 1972, that there is no
possibility of amicable settlement. The Court proceeded to
have the parties submit their evidence for the purpose of the
pre-trial and make admissions for the purpose of pretrial. On
September 25, 1972, the trial court rendered judgment
declaring among others, Carponia T. Ebrado disqualified from
becoming beneficiary of the insured Buenaventura Cristor
Ebrado and directing the payment of the insurance proceeds

2
to the estate of the deceased insured. From this judgment,
Carponia T. Ebrado appealed to the Court of Appeals, but on
July 11, 1976, the Appellate Court certified the case to Us as
involving only questions of law.
ISSUE: Whether or not a common-law wife named as
beneficiary in the life insurance policy of a legally married
man claim the proceeds thereof in case of death of the latter.
HELD: The appealed judgment of the lower court is hereby
affirmed.Carponia T. Ebrado is hereby declared disqualified to
be the beneficiary of the late Buenaventura C. Ebrado in his
life insurance policy. As a consequence, the proceeds of the
policy are hereby held payable to the estate of the deceased
insured. Costs against Carponia T. Ebrado.
A common-law wife named as a beneficiary in the life
insurance policy of a legally married man cannot claim the
proceeds thereof in case the death of the latter. The contract
of insurance is govern by the provisions of the new civil code
on matters not specifically provided for in the insurance code.
Rather, the general rules of civil law should be applied to
resolve this void in the Insurance Law. Article 2011 of the New
Civil Code states: The contract of insurance is governed by
special laws. Matters not expressly provided for in such
special laws shall be regulated by this Code. When not
otherwise specifically provided for by the Insurance Law, the
contract of life insurance is governed by the general rules of
the civil law regulating contracts. And under Article 2012 of
the same Code, any person who is forbidden from receiving
any donation under Article 739 cannot be named beneficiary
of a fife insurance policy by the person who cannot make a
donation to him. Common-law spouses are, definitely, barred
from receiving donations from each other. Also conviction for
adultery or concubinage is not required as only
preponderance of evidence is necessary. In essence, a life
insurance policy is no different from a civil donation insofar as
the beneficiary is concerned. Both are founded upon the same
consideration: liberality. A beneficiary is like a donee, because
the premiums of the policy which the insured pays out of
liberality, the beneficiary will receive the proceeds or profits of
said insurance.
Qua v Law Union. G.R. No. L-4611 December 17, 1955
Facts:
Qua owned 4 warehouses used for the storage of copra and
hemp. They were insured with the Law Union.
Fire broke out and completely destroyed 3 bodegas. The
plaintiff submitted claims totalling P398,562.81. The
Insurance Company resisted payment on the grounds that the
fire had been deliberately caused by the insured or by other
persons in connivance with him.
Que Chee Gan and his brother were tried for arson, but were
acquitted by the trial court. As regards the insurance claim,
the trial court ruled in favor of Qua and entitled him to recover
more than Php 300,000 for indemnities from the insurance
company. Hence, the company appealed to the SC.
In its first assignment of error, the insurance company alleged
that the trial Court should have held that the policies were
avoided for breach of warranty. The contract noted that fire

hydrants were required in a particular measurement of space


(every 150 feet). Hence, they argued that since the bodegas
insured had an external wall perimeter of 500 meters, the
appellee should have 11 fire hydrants in the compound, and
that he actually had only 2, with a further pair.
Issues:
1. WON the insurance company can void the policies it had
issued
2. WON the insured violated the "Hemp Warranty" provisions
of the policy against the storage of gasoline
3. WON the insured planned the destruction of the bodega
Held: No. No. No.
Ratio:
1. The insurer, who at the time of issuance, has knowledge of
existing facts which would invalidate the contract from the
beginning, such constitutes a waiver of conditions in the
contract inconsistent with the facts, and the insurer is stopped
thereafter from asserting the breach of such conditions. Also,
an insurance company intends to executed a valid contract in
return for the premium received; and when the policy contains
a condition which renders it voidable at its inception, and this
result is known to the insurer, it will be presumed to have
intended to waive the conditions and to execute a binding
contract, rather than to have deceived the insured into
thinking he is insured when in fact he is not.
The appellant is barred estoppel to claim violation of the socalled fire hydrants warranty, because it knew the number of
hydrants demanded therein never existed from the very
beginning and issued the policies.
To allow a company to accept one's money for a policy of
insurance which it then knows to be void and of no effect,
though it knows as it must, that the assured believes it to be
valid and binding, is so contrary to the dictates of honesty and
fair dealing, and so closely related to positive fraud, as to the
abhorrent to fair-minded men.
The appellant company so worded the policies that while
exacting the greater number of fire hydrants and appliances,
it kept the premium discount at the minimum of 2 1/2%,
thereby giving the insurance company a double benefit. Such
abnormal treatment of the insured strongly points at an abuse
of the insurance company's selection of the words and terms
of the contract, over which it had absolute control.
Receipt of Premiums or Assessments after Cause for Forfeiture
Other than Nonpayment. It is a well settled rule of law that
an insurer which with knowledge of facts entitling it to treat a
policy as no longer in force, receives and accepts a premium
on the policy, estopped to take advantage of the forfeiture. It
cannot treat the policy as void for the purpose of defense to
an action to recover for a loss thereafter occurring and at the
same time treat it as valid for the purpose of earning and
collecting further premiums.
Moreover, taking into account the well known rule that
ambiguities or obscurities must be strictly interpreted against

3
the party that caused them, the "memo of warranty" invoked
by appellant bars the latter from questioning the existence of
the appliances called for in the insured premises
2. The ambiguity must be held strictly against the insurer and
liberally in favor of the insured, specially to avoid a forfeiture.
So long as insurance companies insist upon the use of
ambiguous, intricate and technical provisions, which conceal
rather than frankly disclose, their own intentions, the courts
must, in fairness to those who purchase insurance, construe
every ambiguity in favor of the insured.

adjusters arrived to conduct an investigation. LU resisted


payment, claiming violation of warranties and conditions,
filing of fraudulent claims, and that the fire had been
deliberately caused by QCG or by other persons in connivance
with him.
QCG, his brother, and some employees were indicted
and tried for arson, but they were acquitted. Thereafter, the
civil suit to collect the insurance money proceeded to its trial.
CFI rendered a decision in QCGs favor.
CFI AFFIRMED; LAW UNION LIABLE

Appellee admitted that there were 36 cans of gasoline in the


building designed. It However, gasoline is not specifically
mentioned among the prohibited articles listed in the socalled "hemp warranty." The cause relied upon by the insurer
speaks of "oils", and is uncertain because, "Oils" usually mean
"lubricants" and not gasoline or kerosene.
If the company intended to rely upon a condition of that
character, it ought to have been plainly expressed in the
policy.
The contract of insurance is one of perfect good faith not for
the insured alone, but equally so for the insurer; in fact, it is
mere so for the latter, since its dominant bargaining position
carries with it stricter responsibility.
Also, the gasoline kept in Bodega No. 2 was only incidental to
his business, being no more than a customary 2 day's supply
for the five or six motor vehicles used for transporting of the
stored merchandise. "It is well settled that the keeping of
inflammable oils on the premises though prohibited by the
policy does not void it if such keeping is incidental to the
business."
3. It was unlikely that Qua burned the warehouse to defraud
the company because he had the resources to pay off the
National Bank in a short time. Also, no motive appears for
attempt to defraud the insurer. While the acquittal of the
insured in the arson case is not res judicata on the present
civil action, the insurer's evidence, to judge from the decision
in the criminal case, is practically identical in both cases and
must lead to the same result, since the proof to establish the
defense of connivance at the fire in order to defraud the
insurer "cannot be materially less convincing than that
required in order to convict the insured of the crime of arson."

On false and fraudulent claims


CFI found that the discrepancies were a result of QCGs
erroneous interpretation of the provisions of the insurance
policies and claim forms, caused by his imperfect English, and
that the misstatements were innocently made and without
intent to defraud. The rule is that to avoid a policy, the false
swearing must be willful and with intent to defraud which was
not the cause.
On the storage of gasoline
Ambiguities or obscurities must be strictly interpreted against
the party that caused them. This rigid application of the rule
has become necessary in view of current business practices.
In contrast to contracts entered into by parties bargaining on
an equal footing, a contract of insurance calls for greater
strictness and vigilance on the part of courts of justice with a
view to protect the weaker party from abuses and imposition,
and prevent their becoming traps for the unwary. The contract
of insurance is one of perfect good faith (uferrimal fidei) not
for the insured alone, but equally so for the insurer; in fact, it
is more so for the latter, since its dominant bargaining
position carries with it stricter responsibility.
QCG admitted that there were 36 cans of gasoline in
Bodega 2. Gasoline is not specifically mentioned among the
prohibited articles listed in the hemp warranty. The cause
relied upon LU speaks of oils. In ordinary parlance, oils
means lubricants and not gasoline or kerosene. The
prohibition of keeping gasoline could have been expressed
clearly and unmistakably.
On fire hydrants warranty

As to the defense that the burned bodegas could not possibly


have contained the quantities of copra and hemp stated in the
fire claims, the insurer relied on its adjuster investigator who
examined the premises during and after the fire. His
testimony, however, was based on inferences from the
photographs and traces found after the fire, and must yield to
the contradictory testimony of those who actually saw the
contents of the bodegas shortly before the fire, while
inspecting them for the mortgagee Bank.

LU is estopped from claiming that there was a violation of


such warranty, since it knew that from the start, the number
of hydrants it demanded never existed, yet it issued policies
and received premiums.

QUA CHEE GAN v. LAW UNION AND ROCK INSURANCE

Sometime in 1979, the Banco Filipino Savings and Mortgage


Bank (respondent) wanted to purchase real properties as new
branch sites for its expansion program. Since the General
Banking Act4 limits a banks real estate holdings to no more
than 50% of its capital assets, the respondents Board of
Directors decided to warehouse some of its existing properties

Qua Chee Gan owned 4 warehouses or bodegas used for the


storage of copra and hemp, which were insured with Law
Union, and the lose made payable to PNB as mortgage of the
hemp and copra. Fire broke out and destroyed bodegas 1, 3
ad 4. QCG informed LU by telegram, and the next day, fire

Ty vs. Banco Filipino Savings and Mortgage Bank (G.R


188302)
FACTS:

4
and branch sites to allow more flexibility in the opening of
branches, and to enable it to acquire new branch sites.
The petitioner, a major stockholder and a director of the
respondent, persuaded two other major stockholders, Pedro
Aguirre and his brother Tomas Aguirre, to organize and
incorporate Tala Realty Services Corporation (Tala Realty) to
hold and purchase real properties in trust for the respondent.
Subsequently, Remedios A. Dupasquier prodded her brother
Tomas to endorse to her his shares in Tala Realty and she
registered them in the name of her controlled corporation,
Add International Services, Inc. The petitioner, Remedios, and
Pedro controlled Tala Realty through their respective
nominees.
In implementing their trust agreement, the respondent sold to
Tala Realty some of its properties. Tala Realty simultaneously
leased to the respondent the properties for 20 years,
renewable for another 20 years at the respondents option
with a right of first refusal in the event Tala Realty decides to
sell
them. However,
in
August
1992,
Tala
Realty
repudiated(renounced) the trust, claimed the titles for itself,
and demanded payment of rentals, deposits, and goodwill,
with a threat to eject the respondent.
The respondent filed 17 complaints against Tala Realty, the
petitioner, Pedro, Remedios, and their respective nominees for
reconveyance of different properties with 17 Regional Trial
Courts (RTCs) nationwide, including Civil Case No. 2506-MN
before Branch 170 of the RTC of Malabon (Malabon case),
subject of the present case.
The petitioner and her co-defendants moved to dismiss the
Malabon case for forum shopping and litis pendentia, citing
the 16 other civil cases filed in various courts involving the
same facts, issues, parties, and reliefs pleaded in the
respondents complaint.
The Malabon RTC denied the motion to dismiss, finding no
commonality in the 16 other civil cases since they involved
different causes of action.

The petitioner filed her separate opposition to the


respondents motion for pre-trial and a motion to hold
proceedings in abeyance, stating that after the dismissal of
G.R. No. 127611, two other similar petitions have been
elevated to this Court: (1) G.R. No. 130184,3involving the CAs
reversal of the dismissal of Civil Case No. Q-95-24830 in the
Quezon City RTC (Branch 91), and (2) G.R. No. 132703. 31
The Malabon RTC granted the motion, and again ordered to
hold proceedings in abeyance.
Six years later, the Malabon RTC directed the parties
counsels to inform it of the status of the pending cases.
In her compliance, the petitioner summarized this Courts
rulings in the consolidated cases of G.R. Nos. 130184 and
139166, and in G.R. No. 132703, and reported on the other
cases involving the same parties decided by this Court, such
as G.R. Nos. 129887, 137980, 132051, 137533, 143263, and
142672, as well as the other related cases decided by this
Court, i.e., G.R. Nos. 144700, 147997, 167255, and 144705.
On the other hand, the respondent filed its compliance with
motion to revive proceedings, citing the Courts consolidated
decision in G.R. Nos. 130184 and 139166, and the decisions in
G.R. Nos. 144700, 167255, and 144705, commonly holding
that there existed no forum shopping, litis pendentia and res
judicata among the respondents reconveyance cases pending
in the other courts of justice.
In her comment to the respondents motion to revive
proceedings, the petitioner argued that the proceedings
should not be revived since all the reconveyance cases are
grounded on the same theory of implied trust which this Court
in G.R. No. 137533 found void for being illegal as it was a
scheme to circumvent the 50% limitation on real estate
holdings under the General Banking Act.
Tala Realty, on the other hand, pointed out that it was the
courts prerogative to suspend or not its proceedings pending
the resolution of issues by another court, in order to avoid
multiplicity of suits and prevent vexatious litigations.

The petitioner filed a motion to hold proceedings in abeyance,


citing the pendency with this Court of G.R. No. 127611 that
assailed the denial of their motion to dismiss Civil Case No.
4521 before the Batangas City RTC (Branch 84), and also
prayed for a writ of prohibition to order the 17 RTC branches
and the three CA divisions, where the same cases were
pending, to desist from further proceeding with the trial of the
cases.

In its May 6, 2008 order, the RTC granted the respondents


motion to revive proceedings, noting that res judicata is not
applicable since there are independent causes of action for
each of the properties sought to be recovered.

The Malabon RTC granted to hold proceedings in abeyance.


When the Malabon RTC denied the respondents motion for
reconsideration, the respondent elevated its case to the CA
via a Rule 65 petition for certiorari. The CA initially dismissed
the petition, but on motion for reconsideration, it modified its
ruling, setting aside the RTCs order to hold proceedings in
abeyance for mootness, due to this Courts dismissal of G.R.
No. 127611 for late filing.

In its March 31, 2009 decision, the CA affirmed the RTCs


orders. It noted that res judicata does not apply since the
issue of validity or enforceability of the trust agreement was
raised in an ejectment case, not an action involving title or
ownership, citing the Courts pronouncement in G.R. No.
144705 that G.R. No. 137533 does not put to rest all pending
litigations involving the issues of ownership between the
parties since it involved only an issue of de facto possession.

The respondent moved for pre-trial. Tala Realty opposed the


motion and filed again a motion to suspend proceedings.

When the CA denied her motion for reconsideration, the


petitioner filed the present petition.

When
the
RTC
denied the
petitioners
motion
for
reconsideration, she elevated her case to the CA via a Rule 65
petition for certiorari, assailing the RTC orders.

5
The petitioner argues that the CA erred in refusing to apply
G.R. No. 137533 under the principle of res judicata by
conclusiveness of judgment and stare decisis, and ignoring
the November 26, 2007 minute resolution in G.R. No. 177865
and the April 7, 2009 consolidated decision in G.R. Nos.
130088, 131469, 155171, 155201, and 16660865 that
reiterated the Courts pronouncement in G.R. No. 137533.
The petition is GRANTED. The assailed decision and resolution
of the Court of Appeals in CA-G.R. SP No. 107104 are
hereby REVERSED and SET ASIDE. Civil Case No. 2506-MN
before Branch 170 of the Regional Trial Court of Malabon,
Metro Manila is hereby DISMISSED.

When the CA denied her motion for reconsideration, the


petitioner filed the present petition.
The petition is GRANTED. The assailed decision and resolution
of the Court of Appeals in CA-G.R. SP No. 107104 are
hereby REVERSED and SET ASIDE. Civil Case No. 2506-MN
before Branch 170 of the Regional Trial Court of Malabon,
Metro Manila is hereby DISMISSED.
ISSUE:
Whether or not the whether the Courts ruling in G.R. No.
137533 applies as stare decisis to the present case.

SUMMARY:

LEGAL BASIS:

Respondents filed 17 complaints against petitioner including


Civil Case No. 2506-MN before Branch 170 of the RTC of
Malabon.

DOCTRINE OF STARE DECISIS, "to adhere to precedents, and


not to unsettle things which are established." Under the
doctrine, when this Court has once laid down a principle of law
as applicable to a certain state of facts, it will adhere to that
principle, and apply it to all future cases, where facts are
substantially the same; regardless of whether the parties and
property are the same. The doctrine of stare decisis is based
upon the legal principle or rule involved and not upon the
judgment, which results therefrom. In this particular sense,
stare decisis differs from res judicata, which is based upon the
judgment.

Petitioner moved for the dismissal of the case in RTC of


Malabon which was denied denied, finding no commonality in
the 16 other civil cases since they involved different causes of
action.
Petitioner filed a motion to hold proceedings in abeyance.
The Malabon RTC granted to hold proceedings in abeyance.

RULING:
Respondents filed for motion for reconsideration but it was
denied by the RTC.
Respondents elevated its case to the CA. The CA initially
dismissed the petition, but on motion for reconsideration, it
modified its ruling, setting aside the RTCs order to hold
proceedings in abeyance for mootness (An issue presenting
no real controversy), due to Courts dismissal of G.R. No.
127611 for late filing.
The respondent moved for pre-trial.
Petitioner opposed the motion and filed again a motion to
suspend proceedings.
The Malabon RTC granted the motion, and again ordered to
hold proceedings in abeyance.
The respondent filed its compliance with motion to revive
proceedings.
The petitioner argued that the proceedings should not be
revived since all the reconveyance cases are grounded on the
same theory of implied trust which this Court in G.R. No.
137533 found void for being illegal as it was a scheme to
circumvent the 50% limitation on real estate holdings under
the General Banking Act.
RTC granted the respondents motion to revive proceedings.
Petitioner filed for motion for reconsideration but it was
denied by the RTC. She elevated her case to the CA via a Rule
65 petition for certiorari, assailing the RTC orders.
The CA affirmed the RTCs orders.

WHEREFORE, the petition is GRANTED. The assailed decision


and resolution of the Court of Appeals in CA-G.R. SP No.
107104 are hereby REVERSED and SET ASIDE. Civil Case No.
2506-MN before Branch 170 of the Regional Trial Court of
Malabon, Metro Manila is hereby DISMISSED.
The 17 cases filed by the respondents have exactly the same
point and the facts are substantially the same and according
to the Principle of Stare Decisis once a case has been decided
one way, any other case involving exactly the same point at
issue, as in the present case, should be decided in the same
manner.

Del Rosario V. Equitable Ins. And Casualty Co., Inc.


(1963)
Lessons Applicable: Ambiguous Provisions Interpreted
Against Insurer
FACTS:
April 13, 1957: Simeon del Rosario, father of the insured who
died from drowning filed a claim for payment with Equitable
Ins. and Casualty Co., Inc. but it refused to pay more than
P1,000 php so a case was filed with the RTC for the P2,000
balance stating that under the policy they are entitled
to P1,000 to P3,000 as indemnity
RTC: entitled to recover P3,000 - policy does not positively
state any definite amount, there is an ambiguity in this
respect in the policy, which ambiguity must be interpreted in
favor of the insured and strictly against the insurer so as to
allow greater indemnity

6
ISSUE: W/N Simeon is entitled to recover P3,000
HELD: YES.
terms in an insurance policy, which are ambiguous, equivocal
or uncertain are to be construed strictly against, the insurer,
and liberally in favor of the insured so as to effect the
dominant purpose of indemnity or payment to the insured,
especially where a forfeiture is involved
reason for this rule is that the "insured usually has no voice in
the selection or arrangement of the words employed and that
the language of the contract is selected with great care and
deliberation by expert and legal advisers employed by, and
acting exclusively in the interest of, the insurance company
Insurance Case Digest: Misamis Lumber Corp. V.
Capital Ins. And Surety Co., Inc. (1966)
Lessons Applicable: Judicial Construction Cannot Alter
Terms
FACTS:
Misamis Lumber Corporation (Misamis), formerly Lanao
Timber Mills, Inc., insured its Ford Falcon motor car
with Capital Insurance & Surety Company (Capital)
November 25, 1961 11 pm: The car broke when it hit a hollow
block lying alongside the water hole which the driver did not
see because the on-coming car did not dim its light
The car was
costing P302.27

towed

and

repaired by

Morosi

petitioner took it upon himself to have the vehicle repaired


without the authority from the insurer, in the policy if such an
event is to occur, and a repair was done without the authority
of the insurer, its liability is limited only to 150 pesos. The
repair
bill
exceeded
such
amount.
Upon filing of claim the insurer refused payment in excess of
150
pesos.
Issue(s):
1.) Can the respondent be made to reimburse the petitioner
for the actual cost of repairs which exceed the repair limit
amount?
Ruling:
The lower courts recourse to legal hermeneutics is not called
for because paragraph 4 of the policy is clear and specific and
leaves no room for interpretation. The interpretation is even
unjustified because it opposes what was specifically
stipulated. Thus it will be observed that the policy drew out
not only the limits of the insurers liability but also the
mechanics that the insured had to follow to be entitled to full
indemnity for repairs. The option to undertake repairs is
accorded to the insurance company per paragraph 2. The said
company was deprived of the option because the insured took
it upon itself to have the repairs made, and only notified the
insurer when the repairs are done. As a consequence,
paragraph 4, which limits the companys liability to P150.00
applies.

Motors

November 29, 1961: After the repairs were made, Misamis


made a report to Capital who only admits liability of P150
CFI: paragraph 4 of the policy is clear and specific and leaves
no room for interpretation that the repair liability is limited to
P150
ISSUE: W/N Misamis is entitled to an amount exceeding P150
HELD: NO.
insurance contract may be rather onerous (one-sided) but that
in itself does not justify the abrogation of its express terms,
terms which the insured accepted or adhered to and which is
the law between the contracting parties
Misamis Lumber vs. Capital Insurance, G.R. L-21380, May 20,
1966
Misamis Lumber vs. Capital Insurance, G.R. L-21380, May 20,
1966

Summary

of

Ruling:

The court ruled in this manner because the policy expressly


limits the liability for authorized repair at P150.00, and being
expressed and unambiguous it leaves no room for
interpretation, and it must be applied as such. The cardinal
principle of insurance law of interpreting insurance contracts
favorably to the insured is applicable only in cases of doubt,
not when the intention of the policy is clear or the language is
sufficiently clear to convey the meaning of the parties,
although the contract may be onerous.
Verendia v. CA - Insurance Policy 217 SCRA 1993
Facts:
> Fidelity and Surety Insurance Company (Fidelity) issued Fire
Insurance Policy No. F-18876 effective between June 23, 1980
and June 23, 1981 covering Rafael (Rex) Verendia's residential
in the amount of P385,000.00. Designated as beneficiary was
the Monte de Piedad & Savings Bank.

Facts:

> Verendia also insured the same building with two other
companies, namely, The Country Bankers Insurance for
P56,000.00 and The Development Insurance for P400,000.00.

The vehicle owned by the petitioner was covered by an


insurance policy issued by the respondent. In such policy it
contained instructions and details on how to proceed with the
claim
for
repairs.

> While the three fire insurance policies were in force, the
insured property was completely destroyed by fire.

When the time came for the vehicle to be repaired, the

7
> Fidelity appraised the damage amounting to 385,000 when
it was accordingly informed of the loss. Despite demands,
Fidelity refused payment under its policy, thus prompting
Verendia to file a complaint for the recovery of 385,000
> Fidelity, averred that the policy was avoided by reason of
over-insurance, that Verendia maliciously represented that the
building at the time of the fire was leased under a contract
executed on June 25, 1980 to a certain Roberto Garcia, when
actually it was a Marcelo Garcia who was the lessee.
Issue:
Whether or not Verendia can claim on the insurance despite
the misrepresentation as to the lessee and the overinsurance.
Held:
NOPE.
The contract of lease upon which Verendia relies to support
his claim for insurance benefits, was entered into between
him and one Robert Garcia, a couple of days after the
effectivity of the insurance policy. When the rented residential
building was razed to the ground, it appears that Robert
Garcia was still within the premises. However, according to
the investigation by the police, the building appeared to have
"no occupants" and that Mr. Roberto Garcia was "renting on
the otherside of said compound" These pieces of evidence
belie Verendia's uncorroborated testimony that Marcelo Garcia
whom he considered as the real lessee, was occupying the
building when it was burned.

Ironically, during the trial, Verendia admitted that it was not


Robert Garcia who signed the lease contract but it was
Marcelo Garcia cousin of Robert, who had also been paying
the rentals all the while. Verendia, however, failed to explain
why Marcelo had to sign his cousin's name when he in fact he
was paying for the rent and why he (Verendia) himself, the
lessor, allowed such a ruse. Fidelity's conclusions on these
proven facts appear, therefore, to have sufficient bases:
Verendia concocted the lease contract to deflect responsibility
for the fire towards an alleged "lessee", inflated the value of
the property by the alleged monthly rental of P6,500) when in
fact, the Provincial Assessor of Rizal had assessed the
property's fair market value to be only P40,300.00, insured
the same property with two other insurance companies for a
total coverage of around P900,000, and created a dead-end
for the adjuster by the disappearance of Robert Garcia.

Basically a contract of indemnity, an insurance contract is the


law between the parties. Its terms and conditions constitute
the measure of the insurer's liability and compliance therewith
is a condition precedent to the insured's right to recovery from
the. As it is also a contract of adhesion, an insurance contract
should be liberally construed in favor of the insured and
strictly against the insurer company which usually prepares it.
Considering, however, the foregoing discussion pointing to the
fact that Verendia used a false lease contract to support his
claim under Fire Insurance Policy, the terms of the policy

should be strictly construed against the insured. Verendia


failed to live by the terms of the policy, specifically Section 13
thereof which is expressed in terms that are clear and
unambiguous, that all benefits under the policy shall be
forfeited "if the claim be in any respect fraudulent, or if any
false declaration be made or used in support thereof, or if any
fraudulent means or devises are used by the Insured or
anyone acting in his behalf to obtain any benefit under the
policy". Verendia, having presented a false declaration to
support his claim for benefits in the form of a fraudulent lease
contract, he forfeited all benefits therein by virtue of Section
13 of the policy in the absence of proof that Fidelity waived
such provision
There is also no reason to conclude that by submitting the
subrogation receipt as evidence in court, Fidelity bound itself
to a "mutual agreement" to settle Verendia's claims in
consideration of the amount of P142,685.77. While the said
receipt appears to have been a filled-up form of Fidelity, no
representative of Fidelity had signed it. It is even incomplete
as the blank spaces for a witness and his address are not filled
up. More significantly, the same receipt states that Verendia
had received the aforesaid amount. However, that Verendia
had not received the amount stated therein, is proven by the
fact that Verendia himself filed the complaint for the full
amount of P385,000.00 stated in the policy. It might be that
there had been efforts to settle Verendia's claims, but surely,
the subrogation receipt by itself does not prove that a
settlement had been arrived at and enforced. Thus, to
interpret Fidelity's presentation of the subrogation receipt in
evidence as indicative of its accession to its "terms" is not
only wanting in rational basis but would be substituting the
will of the Court for that of the parties
PHILIPPINE AMERICAN LIFE INSURANCE COMPANY and
RODRIGO DE LOS REYES vs. HON. ARMANDO ANSALDO,
in his capacity as Insurance Commissioner, and RAMON
MONTILLA PATERNO, JR. G.R. No. 76452 July 26, 1994
Facts
Paterno wrote a letter of complaint to Ansaldo (Insurance
Commissioner) alleging problems encountered by workers and
public consumers as a result of Philamlifes practices. Ansaldo
sought the comment of De Los Reyes [Philamlife President],
who in turn sought a bill of particulars. Paterno responded that
his letter was sufficient in form and sought for a hearing.
Philamlife countered that Paternos response did not enable
him to answer the letter of complaint. Thereafter, a hearing on
the complaint heard the validity of the Contract of Agency
(CoA) complained of, requiring Paterno to supply the specific
provisions in the CoA which he claims to be illegal. Paterno:
(1) reiterated his initial letter of complaint; (2) prayed that the
(a) provisions on charges and fees in the CoA; (b)
implementing provisions in the agents' handbook, agency
bulletins and circulars, be declared null and void; and (3) to
reimburse agents of the amounts deducted from charges and
fees already collected with interest. This was furnished by
Ansaldo to Philamlife who thereafter submitted its
contentions, to wit: (1) Private respondent's letter of August
11, 1986 does not contain any of the particular information
which Philamlife was seeking from him and which he promised
to submit; and (2) [t]hat since the Commission's quasi-judicial
power was being invoked with regard to the complaint, private
respondent must file a verified formal complaint before any

8
further proceedings. Meanwhile, Paterno sought for the
resumption of hearing in re his complaint and about a month
later executed his affidavit. Philamlife [through Manuel
Ortega, SVP] questioned the jurisdiction of Ansaldo and the
locus standi
of Paterno. Ansaldo thereafter notified parties to a hearing.
Philamlife [through Ortega] moved for quashal since
subpoena has no legal basis and Insurance Commission has
no jurisdiction. The same was however denied by Ansaldo.
Hence, this petition.
Issue(s)
(1). Can the Insurance Commission preside over issues
assailing the validity of a Contract of Agency?
Held
(1). No. The general authority of the Insurance Commissioner
is laid down in the Insurance Code, among others, to regulate
the business of insurance. Since a Contract of Agency is not
covered in the authority of the Insurance Commission to
regulate business of insurance, jurisdiction of Ansaldo is
wanting. Ansaldo also has no quasi-judicial power to speak of
in the Insurance Code since xxx his power is limited to claims
and complaints involving any loss, damage or liability for
which an insurer may be answerable under any kind of policy
or contract of insurance xxx". This power does not affect the
relationship between an insurance company and its agents. In
the same light, although the Insurance Code provides for the
subject Insurance Agents and Brokers, the same only speaks
licensing requirements and limitations imposed on insurance
agents and brokers. Thus, it is clear that the Insurance Code
does not grant Ansaldo the authority to take cognizance of the
case. On the other hand, there are two classes of insurance
agents: (1) salaried employees who keep definite hours and
work under the control and supervision of the company; and
(2) registered representatives, who work on commission basis.
The former is cognizable by the Labor Arbiter (as it involves
the Contract of Employment and provisions of the Labor Code)
and the latter by regular courts (as it involves issues on the
Contract of Agency and the Civil Code provisions on Agency).

consideration of the payment by the insured to the company


of the sum included additional premium the Company agrees,
notwithstanding what is stated in the printed conditions of this
policy due to the contrary, that this insurance covers loss or
damage to shock to any of the property insured by this Policy
occasioned by or through or in consequence of earthquake
(Exhs. "1-D", "2-D", "3-A", "4-B", "5-A", "6-D" and "7-C"). In
Exhibit "7-C" the word "included" above the underlined
portion was deleted. On July 16, 1990 an earthquake struck
Central Luzon and Northern Luzon and plaintiffs properties
covered by Policy No. 31944 issued by defendant, including
the two swimming pools in its Agoo Playa Resort were
damaged.

Petitioner advised respondent that it would be making a claim


under its Insurance Policy 31944 for damages on its
properties. Respondent denied petitioners claim on the
ground that its insurance policy only afforded earthquake
shock coverage to the two swimming pools of the resort. The
trial court ruled in favor of respondent. In its ruling, the
schedule clearly shows that petitioner paid only a premium of
P393.00 against the peril of earthquake shock, the same
premium it had paid against earthquake shock only on the two
swimming pools in all the policies issued by AHAC.
Issue: Whether or not the policy covers only the two
swimming pools owned by Gulf Resorts and does not extend
to all properties damaged therein
Held: YES. All the provisions and riders taken and interpreted
together, indubitably show the intention of the parties to
extend earthquake shock coverage to the two swimming pools
only. An insurance premium is the consideration paid an
insurer for undertaking to indemnify the insured against a
specified peril. In fire, casualty and marine insurance, the
premium becomes a debt as soon as the risk attaches. In the
subject policy, no premium payments were made with regard
to earthquake shock coverage except on the two swimming
pools. There is no mention of any premium payable for the
other resort properties with regard to earthquake shock. This
is consistent with the history of petitioners insurance policies
with AHAC.
Philamcare Health Systems, Inc. vs Court of Appeals

Gulf Resorts Inc. vs. Philippine Charter Insurance


Corporation [G.R. No. 156167 May 16, 2005]
Facts: Gulf Resorts is the owner of the Plaza Resort situated at
Agoo, La Union and had its properties in said resort insured
originally with the American Home Assurance Company
(AHAC). In the first 4 policies issued, the risks of loss from
earthquake shock was extended only to petitioners two
swimming pools. Gulf Resorts agreed to insure with Phil
Charter the properties covered by the AHAC policy provided
that the policy wording and rates in said policy be copied in
the policy to be issued by Phil Charter. Phil Charter issued
Policy No. 31944 to Gulf Resorts covering the period of March
14, 1990 to March 14, 1991 for P10,700,600.00 for a total
premium of P45,159.92. the break-down of premiums shows
that Gulf Resorts paid only P393.00 as premium against
earthquake shock (ES). In Policy No. 31944 issued by
defendant, the shock endorsement provided that In

379 SCRA 356 Mercantile Law Insurance Law


Representation Concealment Rescission of an Insurance
Contract Health Care Agreement is an Insurance Contract
In 1988, Ernani Trinos applied for a health care insurance
under the Philamcare Health Systems, Inc. He was asked if he
was ever treated for high blood, heart trouble, diabetes,
cancer, liver disease, asthma, or peptic ulcer; he answered no.
His application was approved and it was effective for one year.
His coverage was subsequently renewed twice for one year
each. While the coverage was still in force in 1990, Ernani
suffered a heart attack for which he was hospitalized. The cost
of the hospitalization amounted to P76,000.00. Julita Trinos,
wife of Ernani, filed a claim before Philamcare for the latter to
pay the hospitalization cost. Philamcare refused to pay as it
alleged that Ernani failed to disclose the fact that he was
diabetic, hypertensive, and asthmatic. Julita ended up paying
the hospital expenses. Ernani eventually died. In July 1990,

9
Julita sued Philamcare for damages. Philamcare alleged that
the health coverage is not an insurance contract; that the
concealment made by Ernani voided the agreement.
ISSUE: Whether or not Philamcare can avoid the health
coverage agreement.
HELD: No. The health coverage agreement (health care
agreement) entered upon by Ernani with Philamcare is a nonlife insurance contract and is covered by the Insurance Law. It
is primarily a contract of indemnity. Once the member incurs
hospital, medical or any other expense arising from sickness,
injury or other stipulated contingent, the health care provider
must pay for the same to the extent agreed upon under the
contract. There is no concealment on the part of Ernani. He
answered the question with good faith. He was not a medical
doctor hence his statement in answering the question asked
of him when he was applying is an opinion rather than a fact.
Answers made in good faith will not void the policy.
Further, Philamcare, in believing there was concealment,
should have taken the necessary steps to void the health
coverage agreement prior to the filing of the suit by Julita.
Philamcare never gave notice to Julita of the fact that they are
voiding the agreement. Therefore, Philamcare should pay the
expenses paid by Julita.

The respondent having become an enemy corporation on


December 10, 1941, the insurance policy issued in its favor on
October 1, 1941, by the petitioner had ceased to be valid and
enforceable, and since the insured goods were burned after
December 10, 1941, and during the war, the respondent was
not entitled to any indemnity under said policy from the
petitioner. However, elementary rules of justice (in the
absence of specific provision in the Insurance Law) require
that the premium paid by the respondent for the period
covered by its policy from December 11, 1941, should be
returned by the petitioner.
GREPALIFE vs. CA
FACTS: A contract of group life insurance was executed
between petitioner Grepalife and DBP. Grepalife agreed to
insure the lives of eligible housing loan mortgagors of DBP.
Leuterio, a physician and a housing debtor of DBP applied for
membership in the group life insurance plan. In an application
form,. Leuterio answered questions concerning his health
condition as follows:
7. Have you ever had, or consulted, a physician for a heart
condition, high blood pressure, cancer, diabetes, lung; kidney
or stomach disorder or any other physical impairment?
Answer: No. If so give details _____________.

FILIPINAS DE COMPANIA DE SEGUROS vs. CHRISTERN,


HUENFELD & CO

8. Are you now, to the best of your knowledge, in good


health?

FACTS:Christern, Huenefeld and Company, a German


company, obtained a fire insurance policy from Filipinas
Compaia for the merchandise contained in a building located
in Binondo, Manila in the sum of P100,000. Filipinas Compaia
is an American controlled company. The building and the
insured merchandise were burned during the Japanese
occupation. Christern filed its claim amounting to P92,650.00
but Filipinas Compaia refused to pay alleging that Christern
is a corporation whose majority stockholders are Germans and
that during the Japanese occupation, America declared war
against Germany hence the insurance policy ceased to be
effective because the insured has become an enemy. Filipinas
Compaia was eventually ordered to pay Christern as ordered
by
the
Japanese
government.

Answer: [x] Yes [ ] NO.

ISSUE:
Whether or not Christern, Huenefeld and Co is entitled to
receive the proceeds from the insurance claim.
HELD:
NO. There is no question that majority of the stockholders of
Christern were German subjects. This being so, Christern
became an enemy corporation upon the outbreak of the war
between the United States and Germany. The Philippine
Insurance Law (Act No. 2427, as amended,) in Section 8,
provides that anyone except a public enemy may be
insured. It stands to reason that an insurance policy ceases
to be allowable as soon as an insured becomes a public
enemy.

Grepalife then issued a Certificate, as insurance coverage of


Leuterio, to the extent of his DBP mortgage indebtedness
amounting to P86,200.00
Later, Leuterio died due to massive cerebral hemorrhage.
Consequently, DBP submitted a death claim to Grepalife.
Grepalife denied the claim alleging that. Leuterio was not
physically healthy when he applied for an insurance coverage.
Grepalife insisted that Leuterio did not disclose he had been
suffering from hypertension, which caused his death.
Allegedly, such non-disclosure constituted concealment that
justified the denial of the claim.
Tthe widow of the. Leuterio, respondent Medarda, filed a
complaint with the RTC, against Grepalife for Specific
Performance with Damages. During the trial, Dr. Mejia, who
issued the death certificate, was called to testify. Dr. Mejias
findings, based partly from the information given by the
respondent widow, stated that Leuterio complained of
headaches presumably due to high blood pressure. The
inference was not conclusive because Leuterio was not
autopsied, hence, other causes were not ruled out.
The trial court rendered a decision in favor of respondent
widow and against Grepalife. The CA sustained the trial
courts decision. Hence, the present petition.
ISSUE:
1. who is the proper party to bring the suit, the widow or the
mortgagee (DBP)?

10
2. WON there was concealment as to justify Grepalifes nonpayment of the insurance proceeds

performed by the mortgagee therein named, with the same


effect as if it had been performed by the mortgagor.

HELD: petition denied

The insured private respondent did not cede to the mortgagee


all his rights or interests in the insurance, the policy stating
that: In the event of the debtors death before his
indebtedness with the Creditor [DBP] shall have been fully
paid, an amount to pay the outstanding indebtedness shall
first be paid to the creditor and the balance of sum assured, if
there is any, shall then be paid to the beneficiary/ies
designated by the debtor. When DBP submitted the
insurance claim against petitioner, the latter denied payment
thereof, interposing the defense of concealment committed by
the insured. Thereafter, DBP collected the debt from the
mortgagor and took the necessary action of foreclosure on the
residential lot of private respondent.

1.

WIDOW

To resolve the issue, we must consider the insurable interest


in mortgaged properties and the parties to this type of
contract.
The rationale of a group insurance policy of mortgagors,
otherwise known as the mortgage redemption insurance, is
a device for the protection of both the mortgagee and the
mortgagor. On the part of the mortgagee, it has to enter into
such form of contract so that in the event of the unexpected
demise of the mortgagor during the subsistence of the
mortgage contract, the proceeds from such insurance will be
applied to the payment of the mortgage debt, thereby
relieving the heirs of the mortgagor from paying the
obligation. In a similar vein, ample protection is given to the
mortgagor under such a concept so that in the event of death;
the mortgage obligation will be extinguished by the
application of the insurance proceeds to the mortgage
indebtedness. Consequently, where the mortgagor pays the
insurance premium under the group insurance policy, making
the loss payable to the mortgagee, the insurance is on the
mortgagors interest, and the mortgagor continues to be a
party to the contract. In this type of policy insurance, the
mortgagee is simply an appointee of the insurance fund, such
loss-payable clause does not make the mortgagee a party to
the contract.
Sec. 8 of the Insurance Code provides:
Unless the policy provides, where a mortgagor of property
effects insurance in his own name providing that the loss shall
be payable to the mortgagee, or assigns a policy of insurance
to a mortgagee, the insurance is deemed to be upon the
interest of the mortgagor, who does not cease to be a party to
the original contract, and any act of his, prior to the loss,
which would otherwise avoid the insurance, will have the
same effect, although the property is in the hands of the
mortgagee, but any act which, under the contract of
insurance, is to be performed by the mortgagor, may be
performed by the mortgagee therein named, with the same
effect as if it had been performed by the mortgagor.
the mortgagee is simply an appointee of the insurance fund,
such loss-payable clause does not make the mortgagee a
party to the contract.
Sec. 8 of the Insurance Code provides:
Unless the policy provides, where a mortgagor of property
effects insurance in his own name providing that the loss shall
be payable to the mortgagee, or assigns a policy of insurance
to a mortgagee, the insurance is deemed to be upon the
interest of the mortgagor, who does not cease to be a party to
the original contract, and any act of his, prior to the loss,
which would otherwise avoid the insurance, will have the
same effect, although the property is in the hands of the
mortgagee, but any act which, under the contract of
insurance, is to be performed by the mortgagor, may be

And since a policy of insurance upon life or health may pass


by transfer, will or succession to any person, whether he has
an insurable interest or not, and such person may recover it
whatever the insured might have recovered, 14the widow of
the decedent Dr. Leuterio may file the suit against the insurer,
Grepalife.
2. The second assigned error refers to an alleged concealment
that the petitioner interposed as its defense to annul the
insurance contract. Petitioner contends that Dr. Leuterio failed
to disclose that he had hypertension, which might have
caused his death. Concealment exists where the assured had
knowledge of a fact material to the risk, and honesty, good
faith, and fair dealing requires that he should communicate it
to the assured, but he designedly and intentionally withholds
the same.
Petitioner merely relied on the testimony of the attending
physician, Dr. Hernando Mejia, as supported by the
information given by the widow of the decedent
On the contrary the medical findings were not conclusive
because Dr. Mejia did not conduct an autopsy on the body of
the decedent. Hence, the statement of the physician was
properly considered by the trial court as hearsay.The CAs
stand is that contrary to appellants allegations, there was no
sufficient proof that the insured had suffered from
hypertension.
Appellant insurance company had failed to establish that
there was concealment made by the insured, hence, it cannot
refuse payment of the claim
The fraudulent intent on the part of the insured must be
established to entitle the insurer to rescind the
contract. Misrepresentation as a defense of the insurer to
avoid liability is an affirmative defense and the duty to
establish such defense by satisfactory and convincing
evidence rests upon the insurer. In the case at bar, the
petitioner failed to clearly and satisfactorily establish its
defense, and is therefore liable to pay the proceeds of the
insurance.
And that brings us to the last point in the review of the case at
bar. Petitioner claims that there was no evidence as to the
amount of Dr. Leuterios outstanding indebtedness to DBP at
the time of the mortgagors death. Hence, for private
respondents failure to establish the same, the action for

11
specific performance should be dismissed. Petitioners claim is
without merit. A life insurance policy is a valued policy. Unless
the interest of a person insured is susceptible of exact
pecuniary measurement, the measure of indemnity under a
policy of insurance upon life or health is the sum fixed in the
policy. The mortgagor paid the premium according to the
coverage of his insurance which states that:
The policy states that upon receipt of due proof of the
Debtors death during the terms of this insurance, a death
benefit in the amount of P86,200.00 shall be paid.
In the event of the debtors death before his indebtedness
with the creditor shall have been fully paid, an amount to pay
the outstanding indebtedness shall first be paid to the
Creditor and the balance of the Sum Assured, if there is any
shall then be paid to the beneficiary/ies designated by the
debtor.
However, we noted that the CA decision was promulgated in
1993. In private respondents memorandum, she states that
DBP foreclosed in 1995 their residential lot, in satisfaction of
mortgagors outstanding loan. Considering this supervening
event, the insurance proceeds shall inure to the benefit of the
heirs of the deceased person or his beneficiaries. Equity
dictates that DBP should not unjustly enrich itself at the
expense of another (Nemo cum alterius detrimenio protest).
Hence, it cannot collect the insurance proceeds, after it
already foreclosed on the mortgage. The proceeds now rightly
belong to Leuterios heirs represented by his widow.
San Miguel Brewery v. Law Union Rock Insurance
Company - Insurance Proceeds 40 PHIL 674
Facts:
> On Jan. 12, 1918, Dunn mortgaged a parcel of land to SMB
to secure a debt of 10T.
> Mortgage contract stated that Dunn was to have the
property insured at his own expense, authorizing SMB to
choose the insurers and to receive the proceeds thereof and
retain so much of the proceeds as would cover the mortgage
debt.
> Dunn likewise authorized SMB to take out the insurance
policy for him.
> Brias, SMBs general manager, approached Law Union for
insurance to the extent of 15T upon the property. In the
application, Brias stated that SMBs interest in the property
was merely that of a mortgagee.
> Law Union, not wanting to issue a policy for the entire
amount, issued one for P7,500 and procured another policy of
equal amount from Filipinas Cia de Seguros. Both policies
were issued in the name of SMB only and contained no
reference to any other interests in the propty. Both policies
required assignments to be approved and noted on the policy.
> Premiums were paid by SMB and charged to Dunn. A year
later, the policies were renewed.
> In 1917, Dunn sold the property to Harding, but no
assignment of the policies was made to the latter.

> Property was destroyed by fire. SMB filed an action in


court to recover on the policies. Harding was made a
defendant because by virtue of the sale, he became the
owner of the property, although the policies were issued in
SMBs name.
> SMB sought to recover the proceeds to the extent of its
mortgage credit with the balance to go to Harding.
> Insurance Companies contended that they were not liable
to Harding because their liability under the policies was
limited to the insurable interests of SMB only.
> SMB eventually reached a settlement with the insurance
companies and was paid the balance of its mortgage credit.
Harding was left to fend for himself. Trial court ruled against
Harding. Hence the appeal.

Issue:
Whether or not the insurance companies are liable to Harding
for the balance of the proceeds of the 2 policies.
Held:
NOPE.
Under the Insurance Act, the measure of insurable interest in
the property is the extent to which the insured might be
daminified by the loss or injury thereof. Also it is provided in
the IA that the insurance shall be applied exclusively to the
proper interest of the person in whose name it is made.
Undoubtedly, SMB as the mortgagee of the property, had an
insurable interest therein; but it could NOT, an any event,
recover upon the two policies an amount in excess of its
mortgage credit.

By virtue of the Insurance Act, neither Dunn nor Harding could


have recovered from the two policies. With respect to
Harding, when he acquired the property, no change or
assignment of the policies had been undertaken. The policies
might have been worded differently so as to protect the
owner, but this was not done.

If the wording had been: Payable to SMB, mortgagee, as its


interests may appear, remainder to whomsoever, during the
continuance of the risk, may become owner of the interest
insured, it would have proved an intention to insure the
entire interest in the property, NOT merely SMBs and would
have shown to whom the money, in case of loss, should be
paid. Unfortunately, this was not what was stated in the
policies.

If during the negotiation for the policies, the parties had


agreed that even the owners interest would be covered by
the policies, and the policies had inadvertently been written in
the form in which they were eventually issued, the lower court

12
would have been able to order that the contract be reformed
to give effect to them in the sense that the parties intended to
be bound. However, there is no clear and satisfactory proof
that the policies failed to reflect the real agreement between
the parties that would justify the reformation of these two
contracts.

Saura Import & Export Co., Inc. V. Philippine


International Surety Co., Inc. (1963)
Lessons Applicable: Mortgagor (Insurance)

Cherie Palileo (debtor-mortgagor) filed a complaint against


Beatriz Cosio (creditor-mortgagee) praying that their
transaction be one of a loan with an equitable mortgage to
secure the payment of the loan. The original counsel of
Cosio Atty. Guerrero being appointed Undersecretary of
Foreign Affairs so she forgot the date of the trial and she was
substituted.
it is a loan of P12,000 secured by a "Conditional Sale of
Residential Building" with right to repurchase. After the
execution of the contract, Cosio insured in her name the
building with Associated Insurance & Surety Co. against fire.

FACTS:

The building was partly destroyed by fire so she claimed an


indemnity of P13,107

Saura Import & Export Co Inc., mortgaged to the Phil. National


Bank, a parcel of land.

Palileo demanded that the amount of insurance proceeds be


credited to her loan

The mortgage was amended to guarantee an increased


amount, bringing the total mortgaged debt to P37,000

RTC: it is a loan with equitable mortgage so the insurance


proceeds should be credited to the loan and refund the
overpayment.

On the land mortgage is a building owned by Saura Import &


Export Co Inc. which was insured with Philippine International
Surety (Insurer) even before the mortgage contract so it was
required to endorse to mortgagee PNB
October 15, 1954: Barely 13 days after the issuance of the fire
insurance policy, the insurer cancelled it. Notice of the
cancellation was given to PNB (mortgagee). But Saura
(insured) was not informed.
April 6, 1955: The building and all its contents
worth P40,685.69 were burned so Saura filed a claim with the
Insurer and mortgagee Bank
RTC: dismissed
ISSUE: W/N Philippine International Surety should be held
liable for the claim because notice to only the mortgagee is
not substantial
HELD:YES. Appealed from is hereby reversed. Philippine
International Surety Co., Inc., to pay Saura Import & Export
Co., Inc., P29,000
It was the primary duty of Philippine International Surety to
notify the insured, but it did not
If a mortgage or lien exists against the property insured, and
the policy contains a clause stating that loss, if any, shall be
payable to such mortgagee or the holder of such lien as
interest may appear, notice of cancellation to the mortgagee
or lienholder alone is ineffective as a cancellation of the policy
to the owner of the property.

ISSUE: W/N Cosio as mortgagee is entitled to the insurance


proceeds for her own benefit
HELD: YES. Modify. collection of insurance proceeds shall not
be deemed to have compensated the obligation of the Palileo
to Cosio, but bars the Cosio from claiming its payment from
the Palileo; and Cosio shall pay to Palileo P810 representing
the overpayment made by Palileo by way of interest on the
loan.
When the the mortgagee may insure his interest in the
property independently of the mortgagor , upon the
destruction of the property the insurance money paid to the
mortgagee will not inure to the benefit of the mortgagor, and
the amount due under the mortgage debt remains
unchanged. The mortgagee, however, is not allowed to retain
his claim against the mortgagor, but it passes by subrogation
to the insurer, to the extent of the insurance money paid
It is true that there are authorities which hold that "If a
mortgagee procures insurance on his separate interest at his
own expense and for his own benefit, without any agreement
with the mortgagor with respect thereto, the mortgagor has
no interest in the policy, and is not entitled to have the
insurance proceeds applied in reduction of the mortgage
debt" But these authorities merely represent the minority
view

Ong Lim Sing V. FEB Leasing And Finance Corp.


(2007)
Lessons Applicable: Existing Interest (Insurance)

liability attached principally the insurance company, for its


failure to give notice of the cancellation of the policy to Saura

FACTS:

it is unnecessary to discuss the errors assigned against


appellee bank

FEB Leasing and Finance Corporation (FEB) leased equipment


and motor vehicles to JVL Food Products with a monthly rental
of P170,494

Palileo V. Cosio (1955)


Lessons Applicable: Mortgagor (Insurance)
FACTS:

13
At the same date, Vicente Ong Lim Sing, Jr. (Lim) an executed
an Individual Guaranty Agreement with FEB to guarantee the
prompt and faithful performance of the terms and conditions
of the lease agreement
JVL defaulted in the payment of the monthly rentals resulting
to arrears of P3,414,468.75 and refused to pay despite
demands
FEB filed a complaint for damages and replevin against JVL,
Lim and John Doe
JVL and Lim admitted the existence of the lease agreement
but asserted that it is in reality a sale of equipment on
installment basis, with FEB acting as the financier
RTC: Sale on installment and the FEB elected full payment of
the obligation so for the unreturned units and machineries the
JVL and Lim are jointly and severally liable to pay
CA: granted FEB appeal that it is a financial lease agreement
under Republic Act (R.A.) No. 8556 and ordered JVL and Lim
jointly and severally to pay P3,414,468.75
ISSUE: W/N JVL and Lim should jointly and severally be liable
for
the
insured
financial
lease

HELD: YES. CA affirmed.


contract of adhesion is as binding as any ordinary contract
The Lease Contract with corresponding Lease Schedules with
Delivery and Acceptance Certificates is, in point of fact, a
financial lease within the purview of R.A. No. 8556
FEB leased the subject equipment and motor vehicles to JVL in
consideration of a monthly periodic payment of P170,494.00.
The periodic payment by petitioner is sufficient to amortize at
least 70% of the purchase price or acquisition cost of the said
movables in accordance with the Lease Schedules with
Delivery and Acceptance Certificates.
JVL entered into the lease contract with full knowledge of its
terms and conditions.
Lim, as a lessee, has an insurable interest in the equipment
and motor vehicles leased.
In the financial lease agreement, FEB did not assume
responsibility as to the quality, merchantability, or capacity of
the equipment. This stipulation provides that, in case of
defect of any kind that will be found by the lessee in any of
the equipment, recourse should be made to the manufacturer.
The financial lessor, being a financing company, i.e., an
extender of credit rather than an ordinary equipment rental
company, does not extend a warranty of the fitness of the
equipment for any particular use. Thus, the financial lessee
was precisely in a position to enforce such warranty directly
against the supplier of the equipment and not against the
financial lessor. We find nothing contra legem or contrary to
public policy in such a contractual arrangement

INSURABLE INTEREST

INSULAR LIFE ASSURANCE COMPANY, LTD. vs.


CARPONIA T. EBRADO
FACTS:
On September 1, 1968, Buenaventura Cristor Ebrado was
issued by The Life Assurance Co., Ltd., on a whole-life for
P5,882.00 with a rider for Accidental Death for the same
amount. He designated Carponia T. Ebrado, his common-law
wife as the revocable beneficiary in his policy. He referred to
her as his wife in the policy. On October 21, 1969, He died as
a result of an accident when he was hit by a failing branch of
a tree. As the policy was in force, the insurance company was
liable to pay the coverage in the total amount of P11,745.73,
representing the face value of the policy in the amount of
P5,882.00 plus the additional benefits for accidental death
also in the amount of P5,882.00 and the refund of P18.00 paid
for the premium due November, 1969, minus the unpaid
premiums and interest thereon due for January and February,
1969, in the sum of P36.27. Carponia T. Ebrado filed a claim
for the proceeds of the Policy as the designated beneficiary
therein, although she admits that she and the insured
Buenaventura C. Ebrado were merely living as husband and
wife without the benefit of marriage. Pascual T. Ebrado, also
filed a claim to the insurance company, this time claiming to
be the legal wife Buenaventura. She asserts that she has a
better right over the proceeds than Carponia who is a
common-law wife. As the insurance company is at a loss as to
whom to give the proceeds, it commenced an action for
interpleader in court. After the issues have been joined, a pretrial conference was held on July 8, 1972, that there is no
possibility of amicable settlement. The Court proceeded to
have the parties submit their evidence for the purpose of the
pre-trial and make admissions for the purpose of pretrial. On
September 25, 1972, the trial court rendered judgment
declaring among others, Carponia T. Ebrado disqualified from
becoming beneficiary of the insured Buenaventura Cristor
Ebrado and directing the payment of the insurance proceeds
to the estate of the deceased insured. From this judgment,
Carponia T. Ebrado appealed to the Court of Appeals, but on
July 11, 1976, the Appellate Court certified the case to Us as
involving only questions of law.
ISSUE: Whether or not a common-law wife named as
beneficiary in the life insurance policy of a legally married
man claim the proceeds thereof in case of death of the latter.
HELD: The appealed judgment of the lower court is hereby
affirmed.Carponia T. Ebrado is hereby declared disqualified to
be the beneficiary of the late Buenaventura C. Ebrado in his
life insurance policy. As a consequence, the proceeds of the
policy are hereby held payable to the estate of the deceased
insured. Costs against Carponia T. Ebrado.
A common-law wife named as a beneficiary in the life
insurance policy of a legally married man cannot claim the
proceeds thereof in case the death of the latter. The contract
of insurance is govern by the provisions of the new civil code
on matters not specifically provided for in the insurance code.
Rather, the general rules of civil law should be applied to
resolve this void in the Insurance Law. Article 2011 of the New

14
Civil Code states: The contract of insurance is governed by
special laws. Matters not expressly provided for in such
special laws shall be regulated by this Code. When not
otherwise specifically provided for by the Insurance Law, the
contract of life insurance is governed by the general rules of
the civil law regulating contracts. And under Article 2012 of
the same Code, any person who is forbidden from receiving
any donation under Article 739 cannot be named beneficiary
of a fife insurance policy by the person who cannot make a
donation to him. Common-law spouses are, definitely, barred
from receiving donations from each other. Also conviction for
adultery or concubinage is not required as only
preponderance of evidence is necessary. In essence, a life
insurance policy is no different from a civil donation insofar as
the beneficiary is concerned. Both are founded upon the same
consideration: liberality. A beneficiary is like a donee, because
the premiums of the policy which the insured pays out of
liberality, the beneficiary will receive the proceeds or profits of
said insurance.

Southern Luzon Employees' Ass. V. Golpeo, Et Al.


(1954)
Lessons Applicable: Invalid Designation
(Insurance)
FACTS:
Roman A. Concepcion listed as his beneficiaries Aquilina
Maloles, Roman M. Concepcion, Jr., Estela M. Concepcion,
Rolando M. Concepcion and Robin M. Concepcion for the death
benefit of an association amounting to P2,505
Two sets of claimants presented themselves:
Juanita Golpeo, legal wife
beneficiaries by the deceased

and

her

children,

named

Marcelino and Josefina Concepcion intervened in their own


right aligning themselves Juanita Golpeo and her minor
children
Elsie Hicban, another common law wife and her child
RTC:Aquilina Maloles and her children the sole beneficiaries
Only the Juanita Golpeo and her minor children and the
intervenors Marcelino and Josefina Concepcion have appealed
to this court

Facts:
> Mrs. Nario applied for and was issued a life Insurance
policy (no. 503617) by PHILAMLIFE under a 20-yr endowment
plant, with a face value of 5T. Her husband Delfin and their
unemancipated son Ernesto were her revocable beneficiaries.
> Mrs. Nario then applied for a loan on the above policy with
PHILAMLIFE w/c she is entitled to as policy holder, after the
policy has been in force for 3 years. The purpose of such loan
was for the school expenses of Ernesto.
> The application bore the written signature and consent of
Delfin in 2 capacities
o

As one of the irrevocable beneficiaries of the policy

o
As father-guardian of Ernesto and also the legal
administrator of the minors properties pursuant to Art. 320 of
the CC.
> PHILAMLIFE denied the loan application contending that
written consent of the minor son must not only be given by
his father as legal guardian but it must also be authorized by
the court in a competent guardianship proceeding.
> Mrs. Nario then signified her decision to surrender her
policy and demand its cash value which then amounted to P
520.
> PHILAMLIFE also denied the surrender of the policy on the
same ground as that given in disapproving the loan
application.
> Mrs. Nario sued PHILAMLIFE praying that the latter grant
their loan application and/or accept the surrender of said
policy in exchange for its cash value.
> PHILAMLIFE contends that the loan application and the
surrender of the policy involved acts of disposition and
alienation of the property rights of the minor, said acts are not
within the power of administrator granted under Art. 320 in
relation to art. 326 CC, hence court authority is required.
Issue:
Whether or not PHILAMLIFE was justified in refusing to grant
the loan application and the surrender of the policy.

ISSUE: W/N Aquilina Molales common-law wife and her


illegitimate children can claim the benefits

Held:

HELD: YES.

YES.

Juanita Golpeo, by her silence and actions, had acquiesced in


the illicit relations between her husband and appellee Aquilina
Maloles

SC agreed with the trial court that the vested interest or right
of the beneficiaries in the policy should be measured on its
full face value and not on its cash surrender value, for in case
of death of the insured, said beneficiaries are paid on the
basis of its face value and in case the insured should
discontinue paying premiums, the beneficiaries may continue
paying it and are entitled to automatic extended term or paidup insurance options and that said vested right under the
policy cannot be divisible at any given time.

new Civil Code recognized certain successional rights of


illegitimate children

Nario v. Philamlife Insurance Company - Loan


Application and Surrender of Policy20 SCRA 434

SC also agreed with TC that the said acts (loan app and
surrender) constitute acts of disposition or alienation of

15
property rights and not merely management or administration
because they involve the incurring or termination of
contractual obligations.
Under the laws (CC and rules of Court) The father is
constituted as the minors legal administrator of the propty,
and when the propty of the child is worth more than P2T (as in
the case at bar, the minors propty was worth 2,500 his
share as beneficiary), the father a must file a petition for
guardianship and post a guardianship bond. In the case at
bar, the father did not file any petition for guardianship nor
post a guardianship bond, and as such cannot possibly
exercise the powers vested on him as legal administrator of
the minors property. The consent give for and in behalf of the
son without prior court authorization to the loan application
and the surrender was insufficient and ineffective and
PHILAMLIFE was justified in disapproving the said applications.
Assuming that the propty of the ward was less than 2T, the
effect would be the same, since the parents would only be
exempted from filing a bond and judicial authorization, but
their acts as legal administrators are only limited to acts of
management or administration and not to acts of
encumbrance or disposition.

Philam v Pineda G.R. No. L-54216 July 19, 1989


Facts:
Pineda procured an ordinary life insurance policy from the
petitioner company and designated his wife and children as
irrevocable beneficiaries.

primary/contingent beneficiary/beneficiaries in this Policy has


been made without reserving the right to change said
beneficiary/ beneficiaries, such designation may not be
surrendered to the Company, released or assigned; and no
right or privilege under the Policy may be exercised, or
agreement made with the Company to any change in or
amendment to the Policy, without the consent of the said
beneficiary/beneficiaries.
The alleged acquiescence of the six (6) children beneficiaries
of the policy cannot be considered an effective ratification due
to the fact that they were minors. Neither could they act
through their father insured since their interests are quite
divergent from one another.
Therefore, the parent-insured cannot exercise rights and/or
privileges pertaining to the insurance contract, for otherwise,
the vested rights of the irrevocable beneficiaries would be
rendered inconsequential.
Of equal importance is the well-settled rule that the contract
between the parties is the law binding on both of them and for
so many times, this court has consistently issued
pronouncements upholding the validity and effectivity of
contracts. Likewise, contracts which are the private laws of
the contracting parties should be fulfilled according to the
literal sense of their stipulations, for contracts are obligatory,
no matter in what form they may be, whenever the essential
requisites for their validity are present
The change in the designation of was not within the
contemplation of the parties. The lower court instead made a
new contract for them. It acted in excess of its authority when
it did so.

He then filed a petition to amend the designation of the


beneficiaries in his life policy from irrevocable to revocable.
Lopez v. Del Rosario44 PHIL 98=

The judge granted the request.


Petitioner promptly filed a motion but was denied. Hence, this
petition.

Issues:
1. WON the designation of the irrevocable beneficiaries could
be changed or amended without the consent of all the
irrevocable beneficiaries.
2. WON the irrevocable minor beneficiaries could give consent
to the change in designation
Held: No to both. Petition dismissed.
Ratio:
Under the Insurance Act, the beneficiary designated in a life
insurance contract cannot be changed without the consent of
the beneficiary because he has a vested interest in the policy.
There was an express stipulation to this effect: It is hereby
understood and agreed that, notwithstanding the provisions of
this policy to the contrary, inasmuch as the designation of the

Facts:
> Benita Del Rosario is the owner of a bonded warehouse in
Manila where copra and other merchandise are deposited.
> Among those who had copra deposited in the warehouse
was Froilan Lopez, the owner of 14 warehouse receipts with a
declared value of P107,990.40 in his name.
> Del Rosario secured insurance on the warehouse and its
contents with 5 different insurance companies in the amount
of P404,800.
> All policies were in the name of Del Rosario, except for one
(with Natl Insurance Co.) for 40T, in favor of Compania Copra
de Tayabas.
> The warehouse and its contents were destroyed by fire.
When Bayne, a fire loss adjuster, failed to effect a settlement
between the Insurance companies and Del Rosario, the latter
authorized Atty. Fisher to negotiate with the Companies.
> An agreement was reached to submit the matter to
arbitration. The claims by different people who had stored
copra in the warehouse were settled with the exception of
Friolan Lopez.

16
> A case was filed in CFI by Lopez. The court awarded him
the sum of P88,492.21 with legal interest.
Issue:
Whether or not Del Rosario acted as the agent of Lopez in
taking out the insurance on the contents of the warehouse or
whether she acted as the reinsurer of the copra.

Henry Harding appealed.


ISSUE:
1. W/N San Miguel has insurable interest as mortgagor only to
the
extent
of
the
mortgage
credit
YES
2. W/N Harding has insurable interest as owner - NO
HELD: affirmed

Held:

section 19 of the Insurance Act:

She acted as the agent of Lopez.

a change of interest in any part of a thing insured


unaccompanied by a corresponding change of interest in the
insurance, suspends the insurance to an equivalent extent,
until the interest in the thing and the interest in the insurance
are vested in the same person

The agency can be deduced from the warehouse receipts, the


insurance policies and the circumstances surrounding the
transaction. Under any aspect, Del Rosario is liable. The law
is that a policy effected by a bailee and covering by its terms
in his own property and property held in trust, inures, in the
event of loss, equally and proportionately to the benefit of all
owners of the property insured. Even if one secured
insurance covering his own goods and goods stored with him,
and even if the owner of the stored goods did not request or
know the insurance, and did not ratify it before the payment
of the loss, it has been held by a reputable court that the
warehouseman is liable to the owner of such stored goods for
his share.

In a case of contributing policies, adjustments of loss made by


an expert or by a board of arbitrators may be submitted to the
court NOT as evidence of the facts stated therein, or as
obligatory, but for the purpose of assisting the court in
calculating the amount of liability.

San Miguel Brewery V. Law Union And Rock Insurance


Co. (1920)
FACTS:
In the contract of mortgage, the owner P.D. Dunn had agreed,
at his own expense, to insure the mortgaged property for its
full value and to indorse the policies in such manner as to
authorize the Brewery Company to receive the proceeds in
case of loss and to retain such part thereof as might be
necessary to satisfy the remainder then due upon the
mortgage debt. Instead, however, of effecting the insurance
himself Dunn authorized and requested the Brewery Company
to procure insurance on the property in the amount of
P15,000 at Dunn's expense.
San Miguel insured the property only as mortgagee.
Dunn sold the propert to Henry Harding. The insurance was
not assigned by Dunn to Harding.
When it was destroyed by fire, the two companies settled with
San Miguelto the extent of the mortgage credit.
RTC: Absolved the 2 companies from the difference. Henry
Harding
is
not entitled to
the
difference
between
the mortgage credit and the face value of the policies.

section 55:
the mere transfer of a thing insured does not transfer the
policy, but suspends it until the same person becomes the
owner of both the policy and the thing insured
Undoubtedly these policies of insurance might have been so
framed as to have been "payable to the San Miguel Brewery,
mortgagee, as its interest may appear, remainder to
whomsoever, during the continuance of the risk, may become
the owner of the interest insured." (Sec 54, Act No. 2427.)
Such a clause would have proved an intention to insure the
entire interest in the property, not merely the insurable
interest of the San Miguel Brewery, and would have shown
exactly to whom the money, in case of loss, should be paid.
But the policies are not so written.
The blame for the situation thus created rests, however, with
the Brewery rather than with the insurance companies, and
there is nothing in the record to indicate that the insurance
companies were requested to write insurance upon the
insurable interest of the owner or intended to make
themselves liable to that extent
If by inadvertence, accident, or mistake the terms of the
contract were not fully set forth in the policy, the parties are
entitled to have it reformed. But to justify the reformation of
a contract, the proof must be of the most satisfactory
character, and it must clearly appear that the contract failed
to express the real agreement between the parties
In the case now before us the proof is entirely insufficient to
authorize reformation.
CHA VS CA
Facts: Petitioner spouses Nilo Cha and Stella Uy-Cha, as
lessees entered into a lease contract with private respondent
CKS Development Corporation as lessor. A stipulation of the
lease contract provides that the Lessee is not allowed to
insure against fire the chattels, merchandise, textiles, goods
and effects placed at any stall or store or space in the leased
premises without first obtaining the written consent and
approval of the Lessor. If the Lessee violates this the policy is
deemed assigned and transferred to the Lessor for his own
benefit.

17
Petitioner took out a policy of fire insurance over the
merchandise inside the leased premises with United Insurance
without consent of CKS.
On the day the lease contract was to expire a fire broke out
inside the leased premises. CKS, wrote a letter to United
asking that the proceeds of the fire insurance be paid directly
to CKS. United refused. Hence, the latter filed a complaint
against the Cha spouses and United.
RTC ruled in favor of CKS. CA affirmed, hence the petition.
Issue: Whether or not CKS can recover from the insurance
policy.
Held: No. Section 18 of the Insurance Code provides that: No
contract or policy of insurance on property shall be
enforceable except for the benefit of some person having an
insurable interest in the property insured.
In the present case, it cannot be denied that CKS has no
insurable interest in the goods and merchandise inside the
leased premises under the provisions of Section 17 of the
Insurance Code: The measure of an insurable interest in
property is the extent to which the insured might be
damnified by loss or injury thereof. Therefore, CKS cannot be
validly a beneficiary of the fire insurance policy taken by
petitioner-spouses. The insurable interest remains with the
Cha spouses.
The stipulation in the lease contract is void for being contrary
to law and public policy. This is in keeping with the provision
under Sec. 25 of the Insurance Code that: Every stipulation in
a policy of Insurance for the payment of loss, whether the
person insured has or has not any interest in the property
insured or that the policy shall be received as proof of such
interest and every policy executed by way of gaming or
wagering is void.

Tai Tong v Insurance G.R. No. L-55397 February 29,


1988

Facts:
Azucena Palomo obtained a loan from Tai Tong Chuache Inc. in
the amount of P100,000.00. To secure the payment of the
loan, a mortgage was executed over the land and the building
in favor of Tai Tong Chuache & Co. Arsenio Chua,
representative of Thai Tong Chuache & Co. insured the latter's
interest with Travellers Multi-Indemnity Corporation for
P100,000.00 (P70,000.00 for the building and P30,000.00 for
the contents thereof)
Pedro Palomo secured a Fire Insurance Policy covering the
building for P50,000.00 with respondent Zenith Insurance
Corporation. On July 16, 1975, another Fire Insurance was
procured from respondent Philippine British Assurance
Company, covering the same building for P50,000.00 and the
contents thereof for P70,000.00.

The building and the contents were totally razed by fire.


Based on the computation of the loss, including the Travellers
Multi- Indemnity, respondents, Zenith Insurance, Phil. British
Assurance and S.S.S. Accredited Group of Insurers, paid their
corresponding shares of the loss. Complainants were paid the
following: P41,546.79 by Philippine British Assurance Co.,
P11,877.14 by Zenith Insurance Corporation, and P5,936.57
by S.S.S. Group of Accredited Insurers Demand was made
from respondent Travellers Multi-Indemnity for its share in the
loss but the same was refused. Hence, complainants
demanded from the other three (3) respondents the balance
of each share in the loss in the amount of P30,894.31
(P5,732.79-Zenith Insurance: P22,294.62, Phil. British: and
P2,866.90, SSS Accredited) but the same was refused, hence,
this action.
In their answers, Philippine British Assurance and Zenith
Insurance Corporation denied liability on the ground that the
claim of the complainants had already been waived,
extinguished or paid. Both companies set up counterclaim in
the total amount of P 91,546.79.
SSS Accredited Group of Insurers informed the Commission
that the claim of complainants for the balance had been paid
in the amount in full.
Travellers Insurance, on its part, admitted the issuance of a
Policy and alleged defenses that Fire Policy, covering the
furniture and building of complainants was secured by a
certain Arsenio Chua and that the premium due on the fire
policy was paid by Arsenio Chua.
Tai Tong Chuache & Co. also filed a complaint in intervention
claiming the proceeds of the fire Insurance Policy issued by
respondent Travellers Multi-Indemnity.
As adverted to above respondent Insurance Commission
dismissed spouses Palomos' complaint on the ground that the
insurance policy subject of the complaint was taken out by Tai
Tong Chuache & Company, for its own interest only as
mortgagee of the insured property and thus complainant as
mortgagors of the insured property have no right of action
against the respondent. It likewise dismissed petitioner's
complaint in intervention in the following words:
From the above decision, only intervenor Tai Tong Chuache
filed a motion for reconsideration but it was likewise denied
hence, the present petition.
Issue: WON Tai Tong had insurable interest
Held: Yes. Petition granted.
Ratio:
Respondent advanced an affirmative defense of lack of
insurable interest on the part of the petitioner that before the
occurrence of the peril insured against, the Palomos had
already paid their credit due the petitioner. However, they
were never able to prove that Tai had a lack of insurable
interest. Hence, the decision must be adverse against them.
However respondent Insurance Commission absolved
respondent insurance company from liability on the basis of

18
the certification issued by the then Court of First Instance of
Davao, Branch II, that in a certain civil action against the
Palomos, Arsenio Lopez Chua stands as the complainant and
not Tai Tong Chuache.
From said evidence respondent commission inferred that the
credit extended by petitioner to the Palomos secured by the
insured property must have been paid. These findings was
based upon a mere inference.
The record of the case shows that the petitioner to support its
claim for the insurance proceeds offered as evidence the
contract of mortgage which has not been cancelled nor
released. It has been held in a long line of cases that when the
creditor is in possession of the document of credit, he need
not prove non-payment for it is presumed. The validity of the
insurance policy taken by petitioner was not assailed by
private respondent. Moreover, petitioner's claim that the loan
extended to the Palomos has not yet been paid was
corroborated by Azucena Palomo who testified that they are
still indebted to herein petitioner.
Public respondent argues however, that if the civil case really
stemmed from the loan granted to Azucena Palomo by
petitioner the same should have been brought by Tai Tong
Chuache or by its representative in its own behalf. From the
above premise, respondent concluded that the obligation
secured by the insured property must have been paid.
However, it should be borne in mind that petitioner being a
partnership may sue and be sued in its name or by its duly
authorized representative. Petitioner's declaration that
Arsenio Lopez Chua acts as the managing partner of the
partnership was corroborated by respondent insurance
company. Thus Chua as the managing partner of the
partnership may execute all acts of administration including
the right to sue debtors of the partnership in case of their
failure to pay their obligations when it became due and
demandable. Public respondent's allegation that the civil case
flied by Arsenio Chua was in his capacity as personal creditor
of spouses Palomo has no basis. The policy, then had legal
force and effect.

Insurance Case Digest: Bachrach V. British


American Assurance Co.
G.R. No. L-5715 December 20, 1910
Lessons Applicable: Effect of Change of Interest in
Thing Insured
FACTS:
E. M. Bachrach insured goods belonging to a general
furniture store, such as iron and brass bedsteads, toilet
tables, chairs, ice boxes, bureaus, washstands, mirrors,
and sea-grass furniture stored in the ground floor and
first story of house and dwelling with an authorized
agent of the British American Assurance Company
British American Assurance Company denied alleging
that:
property covered by the policy to H. W. Peabody & Co.
to secure certain indebtedness due and owing to said
company
interest in certain of the goods covered by the said
policy is trasnferred to Macke to secure certain
obligations assumed by Macke and on behalf of
Bachrach

willfully placed a gasoline can containing 10 gallons of


gasoline close to the insured goods
made no proof of the loss with the time required by the
condition
RTC: British American Assurance Company liable to
bACHRACH
ISSUE: W/N Bachrach can claim
HELD: YES. lower court affirmed
keeping of inflammable oils on the premises, though
prohibited by the policy, does not void it if such
keeping is incidental to the business
It may be added that there was no provision in the
policy prohibiting the keeping of paints and varnishes
upon the premises where the insured property was
stored. If the company intended to rely upon a
condition of that character, it ought to have been
plainly expressed in the policy.
alienation clause - forfeiture if the interest in the
property pass from the insured
there is no alienation within the meaning of the
insurance law until the mortgage acquires a right to
take possession by default under the terms of the
mortgage. No such right is claimed to have accrued in
the case at bar, and the alienation clause is therefore
inapplicable.
we can not find that there is a preponderance of
evidence showing that the plaintiff did actually set fire
or cause fire to be set to the goods in question
It does not positively appear of record that the
automobile in question was not included in the other
policies. It does appear that the automobile was saved
and was considered as a part of the salvaged. It is
alleged that the salvage amounted to P4,000, including
the automobile. This amount (P4,000) was distributed
among the different insurers and the amount of their
responsibility was proportionately reduced. The
defendant and appellant in the present case made no
objection at any time in the lower court to that
distribution of the salvage. The claim is now made for
the first time.
CONCEALMENT
Saturnino v. Philamlife - False Representation 7 SCRA
316
Facts:
> 2 months prior to the insurance of the policy, Saturnino
was operated on for cancer, involving complete removal of
the right breast, including the pectoral muscles and the
glands, found in the right armpit.
> Notwithstanding the fact of her operation, Saturnino did
not make a disclosure thereof in her application for insurance.
> She stated therein that she did not have, nor had she ever
had, among others listed in the application, cancer or other
tumors; that she had not consulted any physician, undergone
any operation or suffered any injury within the preceding 5
years.
> She also stated that she had never been treated for, nor
did she ever have any illness or disease peculiar to her sex,

19
particularly of the breast, ovaries, uterus and menstrual
disorders.
> The application also recited that the declarations of
Saturnino constituted a further basis for the issuance of the
policy.
Issue:
Whether or not the insured made such false representation of
material facts as to avoid the policy.

Argente v. West Coast Life Insurance Co.Misrepresentation 51 PHIL 725


Facts:
> A joint life insurance policy was issued to Bernardo Argente
and his wife Vicenta upon payment of premium, by West
Coast.
> On Nov. 18, 1925, during the effectivity of the policy,
Vicenta died of cerebral apoplexy. Thereafter, Bernardo
claimed payment but was refused.

Held:
YES.
There can be no dispute that the information given by her in
the application for insurance was false, namely, that she
never had cancer or tumors or consulted any physician or
undergone any operation within the preceding period of 5
years.
The question to determine is: Are the facts then falsely
represented material? The Insurance Law provides that
materiality is to be determined not by the event, but solely
by the probable and reasonable influence of the facts upon
the party to whom the communication is due, in forming his
estimate of the proposed contract, or making his inquiries.

> It is admitted that in the Medical Examiners report,


Vicenta, in response to the question asked by the medical
examiner, her replies were as follows:
o How frequently do you use beer, wine, spirits and other
intoxicants? she answered beer only in small quantities.
o What physician have you consulted or been treated by
within the last 5 years and for what illness or ailment? she
answered none
> It is however, not disputed that in 1924, Vicenta was taken
to a hospital for what was first diagnosed as alcoholism and
later changed to manic-depressive psychosis and then again
changed to pscyhonuerosis.
Issue:

The contention of appellants is that the facts subject of the


representation were not material in view of the non-medical
nature of the insurance applied for, which does away with the
usual requirement of medical examination before the policy is
issued. The contention is without merit. If anything, the
waiver of medical examination renders even more material
the information required of the applicant concerning previous
condition of health and diseases suffered, for such information
necessarily constitutes an important factor which the insurer
takes into consideration in deciding whether to issue the
policy or not.
Appellants also contend that there was no fraudulent
concealment of the truth inasmuch as the insured herself did
not know, since her doctor never told her, that the disease for
which she had been operated on was cancer. In the first
place, concealment of the fact of the operation itself was
fraudulent, as there could not have been any mistake about it,
no matter what the ailment.
Secondly, in order to avoid a policy, it is not necessary to
show actual fraud on the part of the insured. In this
jurisdiction, concealment, whether intentional or unintentional
entitled the insurer to rescind the contract of insurance,
concealment being defined as negligence to communicate
that which a party knows and ought to communicate. The
basis of the rule vitiating the contract in cases of concealment
is that it misleads or deceives the insurer into accepting the
risk, or accepting it at a rate of premium agreed upon. The
insurer, relying upon the belief that the insured will disclose
every material fact within his actual or presumed knowledge,
is misled into a belief that the circumstances withheld does
not exist, and he is thereby induced to estimate the risk upon
a false basis that it does not exist.

Whether or not on the basis of the misrepresentations of


Vicenta, Bernardo is barred from recovery.
Held:
YES.
The court found that the representations made by Vicenta in
his application for life insurance were false with respect to her
state of health and that she knew and was aware that the
representations so made by her were false. In an action on a
life insurance policy where the evidence conclusively shows
that the answers to questions concerning diseases were
untrue, the truth or falsity of the answer becomes the
determining factor.

If the policy was procured by fraudulent misrepresentations,


the contract of insurance apparently set forth therein was
never legally existent. It can be fairly assumed that had the
true facts been disclosed by the insured, the insurance would
never have been granted.

Grepalife v. CA 89 SCRA 543


Facts:
> On March 14, 1957, respondent Ngo Hing filed an
application with Grepalife for a 20-yr endowment policy for
50T on the life of his one year old daughter Helen Go.

20
> All the essential data regarding Helen was supplied by Ngo
to Lapu-Lapu Mondragon, the branch manager of GrepalifeCebu. Mondragon then typed the data on the application form
which was later signed by Ngo.
> Ngo then paid the insurance premium and a binding
deposit receipt was issued to him. The binding receipt
contained the following provision: If the applicant shall not
have been insurable xxx and the Company declines to
approve the application, the insurance applied for shall not
have been in force at any time and the sum paid shall be
returned to the applicant upon the surrender of this receipt.
> Mondragon wrote on the bottom of the application form his
strong recommendation for the approval of the insurance
application.
> On Apr 30, 1957, Mondragon received a letter from
Grepalife Main office disapproving the insurance application of
Ngo for the simple reason that the 20yr endowment plan is
not available for minors below 7 yrs old.
> Mondragon wrote back the main office again strongly
recommending the approval of the endowment plan on the
life of Helen, adding that Grepalife was the only insurance
company NOT selling endowment plans to children.
> On may 1957, Helen died of influenza with complication of
broncho pneumonia. Ngo filed a claim with Gepalife, but the
latter denied liability on the ground that there was no contract
between the insurer and the insured and a binding receipt is
NOT evidence of such contract.

Vda. De Canilang v. CA Concealment 223 SCRA 443


(1993)
Facts:
> Canilang consulted Dr. Claudio and was diagnosed as
suffering from "sinus tachycardia." Mr. Canilang consulted the
same doctor again on 3 August 1982 and this time was found
to have "acute bronchitis."
> On the next day, 4 August 1982, Canilang applied for a
"non-medical" insurance policy with Grepalife naming his wife,
as his beneficiary. Canilang was issued ordinary life insurance
with the face value of P19,700.
> On 5 August 1983, Canilang died of "congestive heart
failure," "anemia," and "chronic anemia." The wife as
beneficiary, filed a claim with Grepalife which the insurer
denied on the ground that the insured had concealed material
information from it.
> Vda Canilang filed a complaint with the Insurance
Commissioner against Grepalife contending that as far as she
knows her husband was not suffering from any disorder and
that he died of kidney disorder.
> Grepalife was ordered to pay the widow by the Insurance
Commissioner holding that there was no intentional
concealment on the Part of Canilang and that Grepalife had
waived its right to inquire into the health condition of the
applicant by the issuance of the policy despite the lack of
answers to "some of the pertinent questions" in the insurance
application. CA reversed.

Issue:
Issue:
Whether or not the binding deposit receipt, constituted a
temporary contract of life insurance.

Whether or not Grepalife is liable.

Held:

Held:

NO.

SC took note of the fact that Canilang failed to disclose that


hat he had twice consulted Dr. Wilfredo B. Claudio who had
found him to be suffering from "sinus tachycardia" and "acute
bronchitis. Under the relevant provisions of the Insurance
Code, the information concealed must be information which
the concealing party knew and "ought to [have]
communicate[d]," that is to say, information which was
"material to the contract.

The binding
receipt
in
question
was
merely
an
acknowledgement on behalf of the company, that the latters
branch office had received from the applicant, the insurance
premium and had accepted the application subject for
processing by the insurance company, and that the latter will
either approve or reject the same on the basis of whether or
not the applicant is insurable on standard rates.
Since Grepalife disapproved the insurance application of Ngo,
the binding deposit receipt had never became on force at any
time, pursuant to par. E of the said receipt. A binding receipt
is manifestly merely conditional and does NOT insure
outright. Where an agreement is made between the applicant
and the agent, NO liability shall attach until the principal
approves the risk and a receipt is given by the agent.
The acceptance is merely conditional, and is subordinated to
the act of the company in approving or rejecting the
application. Thus in life insurance, a binding slip or binding
receipt does NOT insure by itself.

The information which Canilang failed to disclose was material


to the ability of Grepalife to estimate the probable risk he
presented as a subject of life insurance. Had Canilang
disclosed his visits to his doctor, the diagnosis made and the
medicines prescribed by such doctor, in the insurance
application, it may be reasonably assumed that Grepalife
would have made further inquiries and would have probably
refused to issue a non-medical insurance policy or, at the very
least, required a higher premium for the same coverage.
The materiality of the information withheld by Canilang from
Grepalife did not depend upon the state of mind of Jaime
Canilang. A man's state of mind or subjective belief is not
capable of proof in our judicial process, except through proof
of external acts or failure to act from which inferences as to
his subjective belief may be reasonably drawn. Neither does

21
materiality depend upon the actual or physical events which
ensue. Materiality relates rather to the "probable and
reasonable influence of the facts" upon the party to whom the
communication should have been made, in assessing the risk
involved in making or omitting to make further inquiries and
in accepting the application for insurance; that "probable and
reasonable influence of the facts" concealed must, of course,
be determined objectively, by the judge ultimately.

death was unrelated to the facts concealed by the insured.


Petitioner's motion for reconsideration was denied. Hence, this
petition.

SC found it difficult to take seriously the argument that


Grepalife had waived inquiry into the concealment by issuing
the insurance policy notwithstanding Canilang's failure to set
out answers to some of the questions in the insurance
application. Such failure precisely constituted concealment on
the part of Canilang. Petitioner's argument, if accepted, would
obviously erase Section 27 from the Insurance Code of 1978.

Held: Yes. Petition dismissed.

Sunlife v CA G.R. No. 105135 June 22, 1995


Facts:
Robert John B. Bacani procured a life insurance contract for
himself from Sunlife. He was issued a policy for P100,000.00,
with double indemnity in case of accidental death. The
designated beneficiary was his mother, Bernarda Bacani.
The insured died in a plane crash. Respondent Bernarda
Bacani filed a claim with petitioner, seeking the benefits of the
insurance policy taken by her son. Petitioner conducted an
investigation and its findings prompted it to reject the claim.
Sunlife informed Bacani that the insured did not disclose
material facts relevant to the issuance of the policy, thus
rendering the contract of insurance voidable. A check
representing the total premiums paid in the amount of
P10,172.00 was attached to said letter.
Petitioner claimed that the insured gave false statements in
his application. The deceased answered claimed that he
consulted a Dr. Raymundo of the Chinese General Hospital for
cough and flu complications. The other questions were
answered in the negative.
Petitioner discovered that two weeks prior to his application
for insurance, the insured was examined and confined at the
Lung Center of the Philippines, where he was diagnosed for
renal failure. During his confinement, the deceased was
subjected to urinalysis tests.
Bernarda Bacani and her husband filed an action for specific
performance against petitioner with the RTC. The court ruled
in favor of the spouses and ordered Sunlife to pay
P100,000.00.
In ruling for private respondents, the trial court concluded that
the facts concealed by the insured were made in good faith
and under a belief that they need not be disclosed. The court
also held that the medial history was irrelevant because it
wasnt medical insurance.
The Court of Appeals affirmed the decision of the trial court.
The appellate court ruled that petitioner cannot avoid its
obligation by claiming concealment because the cause of

Issue: WON the insured was guilty of misrepresentation which


made the contract void.

Ratio:
Section 26 of The Insurance Code required a party to a
contract of insurance to communicate to the other, in good
faith, all facts within his knowledge which are material to the
contract and as to which he makes no warranty, and which
the other has no means of ascertaining.
A neglect to communicate that which a party knows and
ought to communicate, is called concealment.
Materiality is to be determined not by the event, but solely
by the probable and reasonable influence of the facts upon
the party to whom communication is due, in forming his
estimate of the disadvantages of the proposed contract or in
making his inquiries.
The terms of the contract are clear. The insured is specifically
required to disclose to the insurer matters relating to his
health.
The information which the insured failed to disclose were
material and relevant to the approval and issuance of the
insurance policy. The matters concealed would have definitely
affected petitioner's action on his application, either by
approving it with the corresponding adjustment for a higher
premium or rejecting the same. Moreover, a disclosure may
have warranted a medical examination of the insured by
petitioner in order for it to reasonably assess the risk involved
in accepting the application.
Vda. de Canilang v. Court of Appeals- materiality of the
information withheld does not depend on the state of mind of
the insured. Neither does it depend on the actual or physical
events which ensue.
Good faith" is no defense in concealment. The insured's
failure to disclose the fact that he was hospitalized raises
grave doubts about his eligibility. Such concealment was
deliberate on his part.
The argument, that petitioner's waiver of the medical
examination of the insured debunks the materiality of the
facts concealed, is untenable.
Saturnino v. Philippine American Life Insurance " . . . the
waiver of a medical examination [in a non-medical insurance
contract] renders even more material the information required
of the applicant concerning previous condition of health and
diseases suffered, for such information necessarily constitutes
an important factor which the insurer takes into consideration
in deciding whether to issue the policy or not . . . "

22
Anent the finding that the facts concealed had no bearing to
the cause of death of the insured, it is well settled that the
insured need not die of the disease he had failed to disclose to
the insurer. It is sufficient that his non-disclosure misled the
insurer in forming his estimates of the risks of the proposed
insurance policy or in making inquiries as held in Henson.

Fieldmans Insurance v. Songco - Disclosure of Material


Facts in Insurance 25 SCRA 70
Facts:
> In 1960, Sambat, an agent of Fieldmans Insurance,
induced Songco, a man of scant education to enter into a
common carrier insurance contract with Fieldman.
> During the inducement, a son of Songco butted in and said
that they could not accept the type of insurance offered
because theirs was an owner-type jeepney and not a common
carrier.
> Sambat answered that it did not matter because the
insurance company was not owned by the government and
therefore had nothing to do with rules and regulations of the
latter (Fieldman).
> The insurance was executed and approved for a year from
Sept. 1960-1961. It was renewed in 1961 for another year.
> In Oct. 1961, the jeepney collided with a car in Bulacan and
as a result, Sonco died. The remaining members of the family
claimed the proceeds of the insurance with the company but
it refused to pay on the ground that the vehicle was not a
common carrier.
Issue:
Whether or not the Songcos can claim the insurance
proceeds despite the fact that the vehicle concerned was an
owner and not a common carrier.

Kwong Nam applied for a 20-year endowment insurance on


his life for the sum of P20,000.00, with his wife, appellee Ng
Gan Zee as beneficiary. On the same date, Asian Crusader,
upon receipt of the required premium from the insured,
approved the application and issued the corresponding policy.
Kwong Nam died of cancer of the liver with metastasis. All
premiums had been paid at the time of his death.
Ng Gan Zee presented a claim for payment of the face value
of the policy. On the same date, she submitted the required
proof of death of the insured. Appellant denied the claim on
the ground that the answers given by the insured to the
questions in his application for life insurance were untrue.
Appellee brought the matter to the attention of the Insurance
Commissioner. The latter, after conducting an investigation,
wrote the appellant that he had found no material
concealment on the part of the insured and that, therefore,
appellee should be paid the full face value of the policy. The
company refused to settle its obligation.
Appellant alleged that the insured was guilty of
misrepresentation when he answered "No" to the following
question appearing in the application for life insuranceHas any life insurance company ever refused your application
for insurance or for reinstatement of a lapsed policy or offered
you a policy different from that applied for? If, so, name
company and date.
The lower court ruled against the company on lack of
evidence.
Appellant further maintains that when the insured was
examined in connection with his application for life insurance,
he gave the appellant's medical examiner false and
misleading information as to his ailment and previous
operation. The company contended that he was operated on
for peptic ulcer 2 years before the policy was applied for and
that he never disclosed such an operation.

Held:

Issue: WON Asian Crusader was deceived into entering the


contract or in accepting the risk at the rate of premium
agreed upon because of insured's representation?

Yes.

Held: No. Petition dismissed.

The company is estopped from asserting that the vehicle was


not covered. After it had led Federico Songco to believe that
he could qualify under the common carrier liability insurance
policy, and to enter into a contract of insurance paying the
premiums due, it could not thereafter be permitted to change
its stand to the detriment of the heirs of the insured. It knew
all along that Frederico owned a private vehicle. Its agent
Sambat twice exerted the utmost pressure on the insured, a
man of scant education, and the company did not object to
this.

Ratio:

Ng v Asian Crusader G.R. No. L-30685 May 30, 1983


Facts:

Section 27 of the Insurance Law:


Sec. 27. Such party a contract of insurance must
communicate to the other, in good faith, all facts within his
knowledge which are material to the contract, and which the
other has not the means of ascertaining, and as to which he
makes no warranty.
"Concealment exists where the assured had knowledge of a
fact material to the risk, and honesty, good faith, and fair
dealing requires that he should communicate it to the assurer,
but he designedly and intentionally withholds the same."
It has also been held "that the concealment must, in the
absence of inquiries, be not only material, but fraudulent, or
the fact must have been intentionally withheld."

23
Fraudulent intent on the part of the insured must be
established to entitle the insurer to rescind the contract. And
as correctly observed by the lower court, "misrepresentation
as a defense of the insurer to avoid liability is an 'affirmative'
defense. The duty to establish such a defense by satisfactory
and convincing evidence rests upon the defendant. The
evidence before the Court does not clearly and satisfactorily
establish that defense."
It bears emphasis that Kwong Nam had informed the
appellant's medical examiner of the tumor. His statement that
said tumor was "associated with ulcer of the stomach" should
be construed as an expression made in good faith of his belief
as to the nature of his ailment and operation.
While the information communicated was imperfect, the same
was sufficient to have induced appellant to make further
inquiries about the ailment and operation of the insured.
Section 32 of Insurance Law:
Section 32. The right to information of material facts maybe
waived either by the terms of insurance or by neglect to make
inquiries as to such facts where they are distinctly implied in
other facts of which information is communicated.
Where a question appears to be not answered at all or to be
imperfectly answered, and the insurers issue a policy without

any further inquiry, they waive the imperfection of the answer


and render the omission to answer more fully immaterial.
The company or its medical examiner did not make any
further inquiries on such matters from the hospital before
acting on the application for insurance. The fact of the matter
is that the defendant was too eager to accept the application
and receive the insured's premium. It would be inequitable
now to allow the defendant to avoid liability under the
circumstances."

S-ar putea să vă placă și